Using Cauchy Schwartz Inequality (for Integrals)

Click For Summary
The discussion centers on applying the Cauchy-Schwartz Inequality to a specific integral problem involving a function f(t). The user attempts to show that the integral from a to b is bounded by K²(log(b) - log(a)), using the inequality in the context of L² spaces. They set h(t) and g(t) appropriately but encounter difficulties in proving that the integral of t|f(t)|² is less than or equal to K² due to differing limits. The user expresses concern that the problem statement may be flawed, particularly when considering the behavior of f(x) as even or odd functions, which could lead to K approaching 0. The discussion highlights challenges in applying theoretical inequalities to practical scenarios.
SP90
Messages
19
Reaction score
0

Homework Statement



Suppose \int_{-\infty}^{\infty}t|f(t)|dt < K

Using Cauchy-Schwartz Inequality, show that \int_{a}^{b} \leq K^{2}(log(b)-log(a))

Homework Equations



Cauchy Schwartz: |(a,b)| \leq ||a|| \cdot ||b||

The Attempt at a Solution



Taking CS on L^{2} gives us

(|\int_{a}^{b}h(t)g(t)dt|)^{2} \leq (\int_{a}^{b}|h(t)|^{2}dt)(\int_{a}^{b}|g(t)|^{2}dt)

Setting h(t)=\sqrt{t}|f(t)| and g(t)=\frac{1}{\sqrt{t}} we get

(\int_{a}^{b}|f(t)|dt)^{2} \leq (\int_{a}^{b}t|f(t)|^{2}dt)(\int_{a}^{b}\frac{1}{t}dt)

Obviously \int_{a}^{b}\frac{1}{t}dt is log(b)-log(a)

So we just need to show \int_{a}^{b}t|f(t)|^{2}dt \leq K^2

Obviously this is true if \int_{a}^{b}t|f(t)|^{2}dt \leq (\int_{-\infty}^{\infty}t|f(t)|dt)^2

But this is where I get stuck. The limits aren't equal for these two and the Cauchy Schwartz inequality is the wrong way around for them to work.
 
Physics news on Phys.org
your problem statement is probably wrong, since if f(x) is even or odd, K can obviously approach 0, the inequality you want to prove need not hold in general.
 
Question: A clock's minute hand has length 4 and its hour hand has length 3. What is the distance between the tips at the moment when it is increasing most rapidly?(Putnam Exam Question) Answer: Making assumption that both the hands moves at constant angular velocities, the answer is ## \sqrt{7} .## But don't you think this assumption is somewhat doubtful and wrong?

Similar threads

Replies
6
Views
2K
Replies
1
Views
2K
  • · Replies 3 ·
Replies
3
Views
2K
  • · Replies 11 ·
Replies
11
Views
3K
Replies
1
Views
1K
  • · Replies 5 ·
Replies
5
Views
2K
  • · Replies 23 ·
Replies
23
Views
2K
  • · Replies 3 ·
Replies
3
Views
1K
  • · Replies 15 ·
Replies
15
Views
2K
  • · Replies 6 ·
Replies
6
Views
3K